Quickie Questions - Humans - Other Things They Do

Date Answered Questioner (age, location) Question Answer
March 20, 2011keon(canada)how many much energy in watts do you produce per foot step?An adult human metabolism burns about 100 watts = 100 joules/sec. About 22% of that power is converted into mechanical energy, so lets say roughly 20 watts are produced while walking. A foot step takes maybe 0.5 sec of time. So I think the answer is 10 joules per foot step.
March 20, 2011frankI don't know what the "2012 riddle" is. I read that some people believe there will be a global transformation or catastrophe in 2012. Like many predictions, that one may or may not be true, but I find no scientific basis for believing it. Science is concerned with predictions that can be tested by experiment or observation.
March 20, 2011robert(oklahoma/us)Hello. Please help clarify something. A friend of mine recently told me that the brain does not cause or create thoughts, it merely processes them. According to her, the heart has neurons just like the brain, so the heart actually can think. She also asserted that there is growing evidence from cellular memory andRecalling my "Philosophy of mind" course at college, and after just a little catching-up reading, I feel safe saying that too little is known scientifically about thinking to distinguish between "creating" and "processing" thought. It seems obvious that the brain does both, so the onus on your friend is to explain what creates thought, if not the brain? Also, does merely having neurons imply a heart is thinking? A nematode worm has all its 302 neurons mapped by science, but does the worm think?
May 11, 2010joe (minisota)are you a sientistYou can find information about the members of our group in our web page Windows People.
May 4, 2010BernieI am an author and am searching for a good sourcebook that would contain information on the night sky around Denver and the adjacent plains.I personally don't know of such a book, but with Your Sky you can produce maps for any time and date, viewpoint, and observing location. Good luck!
May 4, 2010Adam (NY, USA)Will we oneday attempt to restart Mars' Core to allow mars to rebuild its ecosphere and allow us to live there?Such a project is beyond the knowledge that our civilization has reached. I would even assume that if we were able to do something similar, we would then be able to take better care of our own planet!
May 4, 2010JudithA couple of years ago someone sent me a Powerpoint presentation of the Universe, going outwards to larger views, then coming inwards down to a seed on Earth (I think). If you know of this presentation, please could you let me know the site, or put a link to it on your site?I have seen several of these presentations. I especially like this one, and this one. You can also see a "scale" presentation on our web page A Matter of Scale.
April 26, 2010Amatullah (Pennsyvania)Why do the planets have symbols?The planetary symbols, along with symbols for the Moon, Sun, and even constellations, were developed for use in astronomy and astrology, and are of very ancient origin. Early versions of the planetary symbols were already used in Bianchini's planisphere, from around the 2nd century.
April 23, 2010alice (washington)is it possible to have a thermometer shatter when it gets to hotYes! Be careful when handly thermometers, they can shatter when the temperature rises beyond the thermometer limit.
April 2, 2010deborah (san bernardino, CA)why we can not see the planets move in a telescope.They are very far from us. But you should be able to see their change of position for day to day!
April 1, 2010Albie (India)who was the first scientist? what was his name? what did he find?Some people consider Galileo Galilei to be the first scientist because his spectacular work in the sixteenth century, but Roger Beacon already advocated experimentation in the thirteenth century (one of Bacon's books, Perspectiva (Optics) includes several experiments with all the steps of the scientific method). Yet other people consider Ibn al-Haytham (born in Basra, in what is now Iraq) in 965, to be the first scientist. He wrote more than 200 books and treatises on a wide range of subjects. He was the first person to apply algebra to geometry, founding the branch mathematics known as analytic geometry.
March 26, 2010carl (canada)hi, i was wondering if you could help me i have to do a expermient on the solar system , for a grade 3 science fair can you help meLook, for example, at our A Solar System Model Builders Guide for directions to build a scale model of the solar system.
March 19, 2010Scott (Ohio/United States of America)Is it possible to create a vacuum within a vacuum? Example: a fan works off the principal of creating a vacuum, if you were to start a fan in space what would be teh result.To answr your question we should first try to define what is a vaccum. If you are assuming that a vaccum is a low pressure area, then the answer is affirmative. But for scientists and engineers a vaccum is a region that is devoid of matter. Once such a state is reached, it is not possible to create another vaccum.
March 19, 2010janice (tx)What is Harp? The weather facility in Alaska doing and why?The main goal of HAARP (The High Frequency Active Auroral Research Program, located in Gakona, Alaska) is to further advance our knowledge of the physical and electrical properties of the Earth's ionosphere. This includes studying how the natural ionosphere affects radio signals with the goal of developing techniques that may be available for mitigating these effects to improve the reliability and/or the performance of communication and navigation systems. NOTE: I have heard the various myths around this facility. As it happens, I am an ionospheric scientist, and as so, familiar with this facility. As far as I know, there is no truth in these myths.
March 8, 2010chrissa (usa)the star in bethleham (the year Jesus Christ was born)can we still see that star todayThere is no consensus about the Star of Bethlehem. Modern-day stronomers have proposed various explanations for the star, including a nova, a planet, a comet, an occultation, and a conjunction (gathering of planets). Our page What do you know about the 'Star of Bethlehem' that was mentioned in the New Testament of the Holy Bible? discusses this topic in more detail.
March 3, 2010aneena (bahrain)what do people have stage fright? and self consiousA more appropiate name would be “performance anxiety,” which emphasizes the fear that arises any time we worry about doing well in a high-stakes situation. This article discusses this phenomenon.
February 23, 2010scott (fl)is salt a rockSalt is normally obtained from sea water or rock deposits. Halite is the mineral form of common salt or sodium chloride, NaCl, commonly known as rock salt.
February 23, 2010Sapphire (US)How many humans can fit in the sun?To answer your question let me start with how many planet Earths can fit inside the Sun, and then we will use earth current population to calculate how many humans would fit inside the Sun. The Sun's diameter is about 100 times that of the Earth. This means that 100 Earths can be lined up to stretch across the face of the sun. Now, imagine the the Sun as an empty bowl, and let's try to fill it up with Earths. Now we can stretch the Earths in three directions (up and down, in and out and left and right, and everywhere in between). So, to fill the Sun we would need 100x100x100 = 1 000 000 Earths! Now, our planet current population is estimated by the United States Census Bureau to be 6,804,600,000. Multiplying by the number of Earths that can fit inside the Sun, we get a total of 6,804,600,000,000,000 humans!
February 22, 2010Kris ( Denmark)Is it possible to know exact position of planets and zodiac constellations at any given time? and how is the position measured.Yes, there are many programs available. For example, the Solar System Live gives you the position of the planets in the solar system, and their coordinates (position) in the sky. This page shows the constellations position.
February 17, 2010DIZER (PHILIPPINES)WHAT IS THE COLLISION THEORY?There are many collision theories. For example, collision theory science is a hypothesis, suggested by Max Trautz and William Lewis in 1916 that clearly illustrates how chemical responses take place and why reaction rates vary for various reactions. There is also a collision theory (or Giant Impact theory) for the formation of our moon (the proto-Earth is impacted by a Mars-sized body and Moon formed from the debris).
February 17, 2010Regita (Indonesia)how to use a micrometer?A micrometer (tool) is used for precision measurement. A micrometer is also a metric unit of length equal to one millionth of a meter.
February 5, 2010dan (Virginia)Why doesn't air pressure overcome the air in our bodies, and squish us scince there is a lot more air on top of us than in us?We don’t feel air pressure, because the air and fluids inside our bodies balance the air pressure around us.
February 1, 2010ishinj (sri lanka)what is black planet & is that thrue that the wold ends in 2012In the context of Astronomy I don't know what a black planet is. Regarding 2012, there is no known scientific reason for such a catastrophe to happen.
January 25, 2010Dante (Pennsylvania)Is there an effect on evaporation rates when forming crystals from sugar and sugar substitutes?Yes, once dissolved, sugar is very difficult to re-crystallize because the water hangs on to the sugar molecules.
January 25, 2010Caitlin (New Zealand)Are the eels that i catch from our pond poisonous? we live on a farm so i mean our farm pond.The colour of the eels is a very light brown some times with spots.hope you can reply.....it would be a great help to know if we can smoke them to eat!!For your own safety it would be better if you contact a local biologist to clarify your question. It could be a university department or a government office for wildlife or similar.
January 21, 2010saiannice (florida)how many years on Earth is 800 light years?A light-year is a unit of length. It is defined by the International Astronomical Union (IAU) as the distance that light travels in a vacuum in one Julian year (= 5.88 trillion miles or 9.46 trillion kilometers).
January 21, 2010chrissy (wisconsin)my son wants to know if it has been proven that peoples personalitys changes in a unusual or strange way during a full moon?While superstitions assert that lunar cycles affect human behavior, scientific studies find nothing to support them. This interesting article discusses this topic.
January 18, 2010Tom (Massachusetts)I recall a science teacher (professor?) saying that adding extra weight to the back of a vehicle actually does not improve traction in snow. This makes sense to me because the extra weight would cause more friction between the tires and the snowy or icy road. This friction to overcome in order to achieve forward motion. Thank you and happy holidays to you.And I agree with you. It is obvious, from day-to-day experiences, that heavier vehicles are less prone to slide (although, once out of control, they are harder to stop). In the case of a pick-up truck, with an uneven weight distribution, the addition of extra weight in the back will provide a better traction.
January 15, 2010Lawrence Childress (Texas)How did the Romans / Greeks produce fire?There are some indications that they used hitting stone against stone or flint against metal to generate a spark (the latest is still commonly used nowadays in gas lighters). Often the fire was kept running all day, like in churches or goverment places.
January 8, 2010Anne (India)why is earth's temperature going? and why is global warming taking place?We, humans, are the main culprit. Our web page Global Warming offers lots of information on this topic.
January 8, 2010Ellia (Las Vegas)What factors besides pH affect the corrosion of zinc metal by citrus juices?There is one important factor, how long is the metalic object exposed to the acid.
January 5, 2010josue (california)what is infrastructureInfrastructure can be defined as the basic physical and organizational structures needed for the operation of a society or enterprise.
January 5, 2010Krisha (Florida US)what was the impact caused by the invention of the xray?X-rays were discovered in late 1895 by W. C. Roentgen, a German physicist. Today x-rays are widely used in medicine and security applications (like airports). They are also a great tool for observing celestial bodies.
January 4, 2010manjunathan (INDIA)can we live in other planetOther planets conditions are extremely harsh and not suitable for human life. They can be transformed or space bases can be built at enormous cost, but there are still a long list of question about how such endeavour would affect the human body and mind.
January 4, 2010Patrick (Ghana)do tall buildings resist thunder strike? if yes howYes, they are protected by lightining rods, a metal rod or conductor mounted on top of buildings and electrically connected to ground through a wire.
January 4, 2010jahnavi (india)is there waste in the space?Yes, our aticle Junk in Space! describes this increasing problem.
December 16, 2009Vanessa (nj)Is fluorite a fracture or a cleavageFluorite is a good example of Octahedral Cleavage.
December 16, 2009Shifa (U.A.E)is the world going to finish in 2012?There is not a single scientific fact that supports this catastrophic prediction.
October 27, 2009Jafar (India)What is the exact proportion between land and water on earth?The exact proportion, as you asked, is not an exact science. As it was stated by The encyclopædia of geography by Hugh Murray, William Wallace, Robert Jameson, Sir William Jackson Hooker, Thomas Gamaliel Bradford, William Swainson: "Geographers roundly estimate the ocean and its branches to occupy three fourths of the entire surface of the globe. But to ascertain the exact proportion between the land and water will afford them ample employment for ages to come..."
October 19, 2009rachel (minnesota)why cant radiosondes observe the mesosphere?They can! Just as an example of scientific studies of the mesosphere using radiosondes, see this scientific paper and this one
October 16, 2009Ashvin (Nepal)How do scientists calculate the distance between teo galaxies? Tell me the whole processThere are several methods. The measurement of the disperson of the radiation emited by pulsars, the use of yhe oscillation of Cepheid Variable stars, the parallax method (the relative movement of nearby stars against the very distant stars as the Earth goes in its orbit). This forum discusses several of the methods.
October 13, 2009puvi (malaysia)do legendary creatures exist?Legendary creatures are exactly that, creatures created by human imagination along with legends of their mythical heroes. Dragons, unicorns, mermaids, etc. are fun, but not real!
October 9, 2009Ron (Illinois)What will happen here on earth when all the planets line up ?If your question is related with the 2012 myth, rest assured that no a single scientific fact support such cataclysmic claims. Not all planets will be aligned in 2012, Mercury, Venus and Saturn will be on the other side of the Sun. Furthermore, alignments can't cause earthquakes and volcanic activity (one of the most common claim). The Moon and the Sun's gravity have considerably more effect than any of the other planets on the Earth.
October 9, 2009Jessi (Tonga)Which side is shine? the one with white or the one is black?A shiny surface is one that reflects as much light as possible. If both sides are of the same material and with the same polish, the white side reflects more light.
October 9, 2009timmothy (michigan/ united states)what will happen in 2012 are we going to DIE?No, we are not going to die in 2012. There is no a single scientific fact that supports that idea.
October 6, 2009nathan (china)what causes weeks?A week is a time unit now equal to seven days. In the past, weeks of between 4 and 20 days have been used historically in various places. See this web page and this onefor more information.
October 6, 2009Sanduni (Sri Lanka)How to spot celestial objects in the night sky ?The best way is to use celestial maps. The website SkyMaps.com offers free Sky Maps Each Month, for anybody to explore, learn and enjoy the night sky. there are also many software packages that are very easy to use and to customize to your location.
October 2, 2009Jyoti (U.P. India)On which continent on Earth the Sun shines first in the morning and after that as the Earth moves which are the other continents it cover?It is just a convention adopted long ago. The International Conference in 1884 deemed that there would be a single Universal Day and that this would begin at mean midnight at Greenwich (Meridian 0), the rest of the meridians to the west will have their new day after that.
September 30, 2009Michelle (Kotzebue, Alaska)Found an egg shaped rock, approx 20 lbs and about 1 foot long and 7 inches in heighth, wondering if it would be a metorite rock.It would be better to take the rock to your local natural science museum or university, they will know who to ask.
September 17, 2009Cheyenne (New Zealand)What is a protein?Proteins are one of three major classes of food or source of food energy (4 kcal/gram) abundant in animal-derived foods (ie: meat) and some vegetables. A more scientific definition: Protein is a macromolecule composed of one or more polypeptides, each comprising a chain of amino acids linked by peptide bonds.
September 17, 2009Nicholas (Maryland, United States)If the ocean could be very deep (50 or 100 miles) and a ship sank.Would it go all the way to the bottom or could remain suspended somewhere along that depth due to buyancy of the salt water. (Archinedes) Thank you.Once all the air spaces on the ship has been replaced by water, the ship will sink all the way to the bottom.
September 16, 2009jean (new jersey)i found human figures inside some rocks, i would like to know if it true or my mind is playing tricks on me ?The best bet would be to take the rock to the closes Museum of Natural Sciences or University. They can find a specialist to study the rock.
September 15, 2009angelica (philippines)what is the famous line of nicolaus copernicusThere are many, I especially like: "For I am not so enamoured of my own opinions that I disregard what others may think of them."
August 26, 2009Patti (Philippines)who is marcus valerius martial?Marcus Valerius Martialis was a Latin poet from the Iberian Peninsula best known for his twelve books of Epigrams, published in Rome between AD 86 and 103. You can see his biography here.
August 21, 2009casey (New Jersey)can man reproduce thunderstorms??As far as I know, it is still not possible. Scientists have tried to seed clouds with diverse degrees of success, and to create conditions for lightning. But a full thunderstorm is very complex, and I don’t believe it has been done.
August 21, 2009kierra (ohio)how dose the rock cycle illustrate the conservation of matterDuring the rock cycle the material that forms the rocks is just reused to form new types of rocs. Roks are destroy, but the substance is not!
August 20, 2009Jim (USA)What is the weight, in pounds, of a cubic foot of basalt?Solid basalt is 3011 kilograms per cubic meter, which converts to 188 pounds per cubic foot.
August 20, 2009Benjamin (Texas/U.S.A.)What is the current estimated amount of greenhouse gases generated daily by the 130,000+ commercial airlines flights over the continental U.S.?Although aviation is considered one of the less polutant forms of travel, consider this fact: CO2 emissions from the jet fuel burned per passenger on an average 3200 kilometers (1992 miles) airline flight is about 353 kilograms (776 pounds). I am not aware of a total estimate for all comercial flights.
August 17, 2009Alexander (Saint louis, U.S.A.)How do rockets challenge earth's gravity?Rockets use the energy from its combustble to lift itself (and any load) against the gravity pull. Scientists have calculated very precisely the thrust needed for an object to levae Earth gravitational field.
August 12, 2009miss yuhee (,malaysia)why influenza H1N1 was called by that name?See Influenza A virus nomenclature for a full explanation.
August 10, 2009benjamin (germany)what is space trashSpace debris or orbital debris, also called space junk and space waste, are the objects in orbit around Earth created by humans, and that no longer serve any useful purpose. See this web site for more information.
July 27, 2009Brenna (Iowa)What does UTC stand for?UTC is the Coordinated Universal Time, a time standard based on International Atomic Time (TAI) with leap seconds added at irregular intervals to compensate for the Earth's slowing rotation. An interesting information about the use of UTC (and not CUT) can be found here.
July 27, 2009Kryshana (Georgia)What is the doomsday device and its relevance to the atom smasher and the "secrets of life"?A doomsday device is a theoretical device, usually a weapon, with the potential to destroy all life on Earth, and even Earth itself.
July 22, 2009Emma (England)Do you use a ruler to join the plot points when you are drawing a graph?It depends of what kind of plot I need! For a multipoints plot I would do it by hand, smoothing the curve as I go through the points (there are many mathematical methods to do the same, they are commonly used in all plotting programs).
July 20, 2009Barry (Oregon, USA)If I run my gasoline engine inside the greenouse will that help them, they love Co2.They may love CO2, but your lungs don't! Be carefull with running any combustion engine in an enclosed space.
June 17, 2009Aaron (Kenosha)What is an example of phylum sarcomastigophora?The phylum Sarcomastigophora includes many abundant and ecologically important forms. All are either unicellular or colonial, and may be either autotrophic or heterotrophic . There are two main groups, the flagellates (subphylum Mastigophora) and the amoebas (subphylum Sarcodina). See this website for more information.
June 2, 2009Austin (New Jersey)Do you know nay information about the Aztec gods Tlaloc, Quetzalcoatl, and Huitzilopochtli?Oour website offers information on many mythological figures. Here you can find information about Tlaloc, Quetzalcoatl, and Huitzilopochtli.
June 2, 2009Basauri (Mexico City)¿Cuál es la traducción de lo que dice la PIEDRA ROSETTA?
or "Which is the translation than it says STONE ROSETTA?"
El texto de la Piedra Rosetta es una declaración de Ptolomeo V sobre impuestos y estatuas en templos. El texto completo puede ser consultado aquí.
May 18, 2009Suneet (USA)What causes space junk to fall on to the earth? According to first law of Newton, the objects should continue to remain in motion in space.There are several causes for space junk to fall back to Earth. For example, if due to solar activity we have a geomagnetic storm, the atmosphere would be heated, and would expand. This, in turn, would increase the drag ratio acting on orbiting objects.
May 18, 2009aaron (united kingdom)is the world coming to an end in 2012 because all over the internet it says its coming to an end like more earthquakes and globel warming and all that stuff and the asteroid.There is no scientific reason for such a disaster to happen.
May 18, 2009Nitya (California)What are th advantages and disadvantages of bringing Martian rock to Earth to look for organisms?Without being completely sure of what your question is about, I can easily imagine a couple of pros and cons. PROS: Better equipment, bigger team of scientists. CONS: possibility of contamination of the sample, absence of the original environment.
May 7, 2009DEMILADE (CANADA)why can't one use a ballpoint pen in spaceActually this is a myth. Read this extract of the Astronaut Pedro Duque's Soyuz/ISS Diary for his experience with a normal ballpoint pen in space.
April 30, 2009andrew (australia)how old are our atoms in our body?In general the question can be asked as: How old are atoms? Please, visit this page for an interesting discusson on this topic.
April 30, 2009monika (india)what is pollution? i want to make a project based on pollution.for class 11 .plz help me & give me tips for good result.now plz...............Pollution is defined as undesireable change in the physical, chemical, or biological characteristics of the air, water, or land that can harmfully affect the health, survival, or activities of human or other living organisms.
April 27, 2009Joe (Ayrshire, Scotland)When a spaceship goes into outer space why does the earth not fly off into the distance, since it is travelling at around 250,000mph round the sun?Because althought the force acting on both bodies (earth - space ship) is the same, the acceleration that each one gets is inversely proportional to the mass. The mass of Earth is substantially bigger than the mass of the rocket, and the resultant acceleration is very close to zero.
April 27, 2009jennifer (CA)im conducting an experiment my topic is
"difference between female and male n indurance running"
my quietion is
how would i be able to test someones fitness ??
For a middle/high scool level project, the simplest way would be to measure heart rate before and after an exercise, and express the results as fractions of heart rate after excerse/heart rate before exercise. A fitter persone would have a smaller increase on heart rate after the exercise.
April 20, 2009jazmine (utah)should we continue the exploration of the moon?Yes, I believe so, but without stopping the support to other fields of study, including the advancing of the study of our own Earth.
April 17, 2009jamie (canada)why do scientists study about rocks,minerals and thelayersof the earth?There are many reason, from simple scientific curiosity, to the need to know our planet to make better use of its resources, and the possibility to predict natural disasters.
April 16, 2009Amia (USA)How does dew get on the grass in the early morning,like what gets condensed?During the day, water accumulates in the air as vapor. At night, when the temperature decreases, the air capacity to hold water vapor diminishes up to the "dew point", whn the atmospheric water condenses and form the dew.
April 15, 2009jess (canada)my science teacher told us that "the moon" and "the earth" aren't really the names but that it should be earth and moon is that true????? he compared it to say if you lived on mars you wouldn't saty i live on "the mars" you would just say i live on marsIt is just a semantic consideration. As a curiosity, in Spanish it is used in the same way: La Tierra, La Luna.
April 14, 2009warisha (pakistan)why poulition is increasing on our earth?One of the main pollutants is the CO2 emited by cars' and trucks' tailpipe, as a result of the combustion of fosil combustibles, a relatively recent technology..
March 25, 2009brandon (united states Al)what is the deinition for river bedA stream or river bed is defined as the channel bottom of a stream or river or creek; the physical confine of the normal water flow.
March 24, 2009Alyssa (Virginia)What is the name of the scale used to measure a hurricane?The Saffir-Simpson Hurricane Scale is the name of the system used to measure the intensity of these massive storms.
March 23, 2009muthu (india)electrons are always revolve around a nucleus but what happened when it is involving in covalent bonding or some other bond?In general, a covalent bond is a form of chemical bonding that is characterized by the sharing of pairs of electrons between atoms. See this page for a simple description of this bond.
March 16, 2009Alfredo (Mexico)What is Punnett's square?A Punnett square is a chart which shows/predicts all possible gene combinations in a cross of parents whose genes are known. See this page for more information.
March 16, 2009Cody (Ohio)what is the difference between pyrite and gold?There are some clear differences. Gold is heavy, does not shine (does not reflect the sunlight). It is gold in color, and dull. Pyrite, on the other hand, is not heavy, it shines, and is yellow in color.
March 10, 2009Bilal (Canada, Saskatoon, SK)while the hubble space telescopecan take pictures many light years away why can't it take a picture of gliese 581 c which is only 20.5 light years away???The optical observations of exoplanets is not an easy task. The bright and mass of the star in the system makes difficult the direct observation. Other instruments, like the High Accuracy Radial Velocity for Planetary Searcher (HARPS) are used. HARPS is a spectrographic instrument that splits light to find wobbles in different wavelengths. Those wobbles can reveal the existence of other worlds.
March 4, 2009Johnna (America)What is the difference between Titans and gods?The Titans, also known as the elder gods, ruled before the Olympians overthrew them. The ruler of the Titans was Cronus who was de-throned by his son Zeus.
March 2, 2009Andreas (Sweden)How does a nuclear winter affect the climate and what creatures would become extinct?
Would the oxygen levels drop/how much?
How long would it last?
Nuclear winter describes the predicted climatic effects of nuclear war. It is also used in a similar way for the effects of an asteroid impact or a supervolcano explosion. In any of these cases, severely cold weather and reduced sunlight for a long period of time (months or years) would be caused by the large amounts of smoke and soot injected into the Earth's stratosphere due to the detonation of large numbers of nuclear weapons. Such escenario has not happened in recorded history, but there is no doubt that the devastation, due to cold temperatures and lack of sunlight, of the main agricultural areas would have a huge impact in the human population.
February 27, 2009JUANITA (NEWJERSEY/U.S.A.)HOW CAN LATITUDE AND LONGTITUDE LINE BE HELP AND WHO USES THEMEveryone using a map need to know and understand the latitud/longitude system. It permits to locate a specific point on a map.
February 24, 2009Hargobind (St.Louis, Missouri)How could it be possible to have humans be living on Mars in 2009I believe we are very far from having a human colony anywhere outside our planet, with the only exception of the International Space Station.
February 20, 2009devon (Canada)what are we doing that is causing greenhouse gases??????The main sources of greenhouse gases due to human activity are: burning of fossil fuels and deforestation leading to higher carbon dioxide concentrations. Land use change (mainly deforestation in the tropics) account for up to one third of total anthropogenic CO2 emissions.[20] livestock enteric fermentation and manure management,[22] paddy rice farming, land use and wetland changes, pipeline losses, and covered vented landfill emissions leading to higher methane atmospheric concentrations. Many of the newer style fully vented septic systems that enhance and target the fermentation process also are sources of atmospheric methane. use of chlorofluorocarbons (CFCs) in refrigeration systems, and use of CFCs and halons in fire suppression systems and manufacturing processes. agricultural activities, including the use of fertilizers, that lead to higher nitrous oxide (N2O) concentrations. From Greenhouse gas.
February 19, 2009Kaylan (Texas)Is a Dalmation Stone a sedimentary, igneous or metamorphic?Dalmatian Stone, or orbicular Rhyolite, is an igneous rock.
February 10, 2009Darcie (Australia)why cant we see Jupiters moons?Using telescopes we can!
February 9, 2009leny (philippines)who is the founder and father of trigonometry???The Greek mathematician Hipparchus is condidered to be the founder of trigonometry. He produced the first known table of chords in about 140 BC.
February 8, 2009cya (india)sir,can you please give some of the sites related to science including astronomy?I would suggest that you start with our website Windows to the Universe. We cover many aspect of human knowledge, including science and astronomy, and also offer many links to other sites.
February 5, 2009sudiang (philippines)what is the control measures of green hous effectThere are many measures being adopted to slow down the green house effect. Some of them are of big scale, some others can be done at our own house. Here are some tips: 1.Car pool or find another way to travel in order to preserve the earth's natural resources. 2.Reuse house hold items. That way our land fills don't fill so quickly. 3.Compost your left over fruit and vegetable parts. It can give you rich soil and also reduces the amount that goes into the landfills. 4.Don't sit around the house wasting electricity when you can be outdoors and playing in the sun. 5. Recycle your paper, beverage containers, milk jugs, and tin cans so that they can be used to make new material and so that we don't waiste natural resources.
February 5, 2009Zania Avalos (California)I heard that in 30 more years there will be a bigger earthquake than ever and alot of people might die,is that true?Scientists are still working toward a consistently succesful prediction of earthquakes. It is currently very hard to predict seismic activity for such a long time.
February 4, 2009Alex (Maine, USA)When Mt. St. Helens erupted, scientists dated the new rocks formed from the eruption using the Radiometric dating systom. Although the rocks were less than 20 years old, the dating systom said they were over 2000000! Is the Radiometric dating system really reliable? If it is not, how do we know how old the universe is?All of the methods under the definition of Radiometric Dating are very accurate. This detailed article offers a good review of the methods in a clear language.
February 4, 2009William (NY, USA)I believe I have found a fossilised rock with organisms imbibed in it. This rock was found on a small beach in Newfoundland. How do I find out more about it? ThanksThe easier way is to approach your local Museum of Natural Sciences.
February 3, 2009Viola (United States)I am studying about Autumnal Equinoxes and cannot find out when the year of 2008's will be. Can you guys help me out?Here you can find a table with the equinoxes and solstices from 2000 to 2020.
January 29, 2009Asif (Pakistan)if a hellicopter is suspended in air and earth rotates about its axis for 12 hours, where will the hellicopter land after 12 hrs,new position or old position and why?There are many factors to keep in mind in order to offer an exact answer to this question. Most importantly the condition of the atmosphere at this particular place. Where there winds? Was it during the night, day, a combination? How high is the hellicopter suspended? At what latitud? Season? etc.
January 28, 2009cya (india)why do humans have wrinkles on their skin when they are old?The two basic organic reasons for the degradation of our skin as we age because are that we lose the two main building blocks for skin health, collagen and elastin. Of course, there are external factors that could accelerate this process. From gravity (which we can't do anything about) to a long list of other causes that we can, up to certain point, control: sun damage, smoking, dehydration, diet, lack of sleep,etc.
January 28, 2009kajal (india)Sir if i have to become an astronaut what should be my i q and what knowledge should i gain at my age?This is a real example of NASA requirements:
NASA National Aeronautics and Space Administration
Lyndon B. Johnson Space Center Houston, Texas
Announcement for Mission Specialist and Pilot Astronaut Candidates
Astronaut Candidate Program Basic Qualification Requirements
Applicants must meet the following minimum requirements prior to submitting an application.
Mission Specialist Astonaut Candidate:
1. Bachelor's degree from an accredited institution in engineering, biological science, physical science, or mathematics. Degree must be followed by at least three years of related progressively responsible, professional experience. An advanced degree is desirable and may be substituted for part or all of the experience requirement(master's degree= 1 year, doctoral degree= 3 years). Quality of academic preparation is important. 2. Ability to pass a NASA class II space physical, which is similar to a civilian or military class II flight physical and includes the following specific standards: Distant visual acuity: 20/150 or better uncorrected, correctable to 20/20, each eye. Blood Pressure: 140/90 measured in sitting position. 3. Height between 58.5 and 76 inches.
Pilot Astronaut Candidate:
1. Bachelor's degree from an accredited instution in engineering, biological science, physical science, or mathematics. Degree must be followed by at least three years of related progressively responsible, professional experience. An advanced degree is desirable. Quality of academic preparation is important. 2. At least 1000 hours pilot-in-command time in jet aircraft. Flight test experience highly desirable. 3. Ability to pass a NASA class I space physical which is similar to a civilian or military class I flight physical and includes the following specific standards: Distant visual acuity: 20/50 or better uncorrected, correctable to 20/20, each eye. Blood pressure: 140/90 measured in sitting position. 4. Height between 64 and 76 inches.
Citizenship Requirements. Applicants for the Astronaut Candidate Program must be citizens of the United States. Note on Academic Requirements Applicants for the Astronaut Candidate Program must meet the basic education requirements for NASA engineering and scientific positions -- specifically: successful completion of standard professional curriculum in an accredited college or university leading to at least a bachelor's degree with major study in an appropriate field of engineering, biological science, physical science, or mathematics. The following degree fields, while related to engineering and the sciences, are not considered qualifying: -Degrees in technology(Engineering Technology, Aviation Technology, Medical Technology, etc.) -Degrees in Psychology(except for Clinical Psychology, Physiological Psychology, or Experimental Psychology which are qualifying) -Degrees in Nursing -Degrees in Social Sciences(Geography, Anthropology, Archaeology, etc.) -Degrees in Aviation, Aviation Management, or similar fields.
January 23, 2009elspeth (Australia QLD)what does it mean when you write about a moon's diameter and write the number times another number, for example (Telesto) 30X16let's say that you have the mass of our planet expressed as 5.98 x 1024 kilograms. This is call scientific notation. It means that the number up front (in this case 5.98) should be multiplied by a 1 followed by 24 zeroes. The result in this particular case will be: 5,980,000,000,000, 000, 000, 000,000 kilograms (Wouldn't you agree that it is easier to express such a large number in scientific notation?)
November 12, 2008quincy (philippines)what is gradation in geological term?The distribution of the various sized particles that constitute a sediment, soil, or other material.
November 11, 2008nelly (new york)THREE FACTORS THAT AFFECT OUR CLIMATERegional or global climate can be affected by many anthropogenic and natural causes. A violent volcanic eruption, or meteorite hit, can produce enough atmospheric contamination to block the sunlight. The burn of forests (natural or man-made) can also have an important role. The release of industrial contaminants (including the automobiles tail-pipe gases) to the atmosphere is maybe one of the most important factors to consider.
November 6, 2008Marc (United States of America)How come we can not tell when a earthquake happens?There are so many known (and maybe many other unknown) factors related with earthquakes that scientists have not been succesful predicting earthquakes, but serious efforts are being made, and there is the real possibility that soon we will be able to predict them.
November 6, 2008Marc (North Caronlina United States of America)How come we can not tell when a earthquake happens?There are so many known (and maybe many other unknown) factors related with earthquakes that scientists have not been succesful predicting earthquakes, but serious efforts are being made, and there is the real possibility that soon we will be able to predict them.
November 6, 2008ANYN ( Arizona)When was glass first made by man?It is not well known where and when the first man-made glass was produced but it is thought to be in Mesopotamia (located today in the Iraq/Syria area) between 2000 and 3000 B.C. as indicated by written records found in this area. More information here.
November 6, 2008ashley (united states)what is the study of the entire atmosphere called?Aeronomy is a branch of science that deals with the atmosphere of the Earth and the other planets with reference to their chemical composition, physical properties, relative motion, and responses to radiation from space.
November 5, 2008EmTa (USA)What can kids do to stop global warming?WE ALL NEED FIRST TO EDUCATE OURSELVES ON THIS TOPIC. And we also need to educate our parents and friends. There are many things to do, from producing less trash and using fluorescent lamps, to walk or ride more our bikes instead of drive to everywhere. In this article you can find 50 Things To Do To Stop Global Warming.
November 5, 2008abhinav ( india)what is the mechanism LARGE HADRON COLLOIDER?The Large Hadron Collider (LHC) is a huge (world's largest) high-energy particle accelerator, intended to collide opposing beams of protons or lead ions, moving at approximately 99.999999% of the speed of light. Please, visit its website for more information: LHC.
November 3, 2008Sarthak (India)Why are the star images larger for brighter stars?Because there is more light to be collected by the lenses of the telescopes.
October 22, 2008AlvynWhat mathematics was used to determine the very deep sounds produced by Black Holes?It seems to be from analysis of pictures taken by the Chandra X-ray Observatory. Quoting from this article: "In 2002, astronomers obtained a deep Chandra observation that shows ripples in the gas filling the cluster. These ripples are evidence for sound waves that have traveled hundreds of thousands of light years away from the cluster's central black hole."
October 21, 2008Linnea (USA)I've been thinking of universal time and distance measurements and I am curious if this has not already established, has it? What I mean is a standard which is consistent throughout the universe, ie like a unit of distance = Planck Length, but how can time be represented universally?Universal Time (UT) is a timescale based on the rotation of the Earth. There are several versions of it, the most commonly used being UTC and UT1 (see this link). Now, for a nice discussion of how time applies to the universe, see this article.
October 16, 2008cya (india)can't we send a satellite in space and let it wander over there with a video recorder? by this we will know a lot about space and discover new planets,moons,stars,galaxies,aliens and lots more and we would even (at least to some extint) that how huge is universe. while using a video recorder we would have to use a lot of tape. and sir,can you please that how bubble gums (chewing gums) are prepared with poly(styrene-butadine)and why do they stick to our skin and not to our teeth. and sir, can you please also tell if there are aliens in the dragon's triangle in pacafic ocean or if aliens ever came to area 51 in U.S.A???Something similar is already being done. All satellites have sensors that observe and collect information about our planet, the sun, and the space, and then it is send to Earth.
October 9, 2008shruti (india)why is the flow of electric current opposite to the flow of electronsThis is an old discussion with many sides. If an electric current is invariably made of negatively-charged particles (electrons), the your statement is true, but this is not always the case! There are instances when electric currents can really be a flow of positive particles.
October 9, 2008candice (new zealand)is there anything currently happening to prevent the artic ice melting and disrupting the ocean conveyor belt??Yes, actions are being taken to prevent the effects of the climate changes. International agreements to reduce the use of greenhouse gases are in effect, the development of new techniques to reduce the amount of dangerous gases from autos and industry are already showing results (for example, catalytic converters, hybrid cars, etc.), the use of solar and wind energy is spreading, etc. But what I think is the major accomplishment is that humanity has come to realize that we cannot keep abusing of our home planet as until now.
October 8, 2008cya (india)is it true that a black hole can be formed by a 27 kilometre long machine if the the protons are being crashed at the speed of light?This is not true. This experiment, known as article offers a nice review of the topic.
October 6, 2008Frank (The State of Indiana)Is there any treatment or process which an organic material, such as wood or other plant fibers, can undergo which would render it comparable in strength and hardness to rival Bronze or Iron Age tools which does not involve a modern process, such as the application of sophisticated resins?
I know this seems like a strange question, but it's for a book I'm writing. Finding a "yes" answer to this question would really be a boon, but my own research hasn't yielded any results. The problem is I don't know enough about physics or chemistry to figure it out on my own.
During the Bronze Age techniques for smelting copper and tin from naturally-occurring outcroppings of copper and tin ores were developed, which allowed the creation of a bronze alloy by melting those metals together, and casting them into bronze artifacts. I (personally) don't believe there is a simple method to obtain similar results with organic materials.
October 3, 2008Hasmin (Philippines)who introduce the system of classifying or grouping living things?Linnaeus developed a Binomial Nomenclature (two name system) in which each organism has a genus (first) and species (second) names.
September 30, 2008andrea (pittsburgh)what time in history did mythology startThere is no particular time for the begining of mythology. Different cultures described what they saw under their own terms, frequenly relating the shapes they saw on the sky with their particular gods.
September 30, 2008alaina (North Carolina, U.S)In what order does the nitrogen cycle go in?Nitrogen is a part of living things like plants and animals. Nitrogen is also a part of non-living things like the air above and the dirt below. Nitrogen does not stay in one place. It moves slowly between living things, dead things, the air, soil and water. These movements are called the nitrogen cycle. More information on our web page The Nitrogen Cycle.
September 29, 2008cya (india)sir, can you please tell how many marks do we have to bring to be an astronaut? and what should be our I.Q? and how many hours do we have to study?This is a real example of NASA requirements:
NASA National Aeronautics and Space Administration
Lyndon B. Johnson Space Center Houston, Texas
Announcement for Mission Specialist and Pilot Astronaut Candidates
Astronaut Candidate Program Basic Qualification Requirements
Applicants must meet the following minimum requirements prior to submitting an application.
Mission Specialist Astonaut Candidate:
1. Bachelor's degree from an accredited institution in engineering, biological science, physical science, or mathematics. Degree must be followed by at least three years of related progressively responsible, professional experience. An advanced degree is desirable and may be substituted for part or all of the experience requirement(master's degree= 1 year, doctoral degree= 3 years). Quality of academic preparation is important. 2. Ability to pass a NASA class II space physical, which is similar to a civilian or military class II flight physical and includes the following specific standards: Distant visual acuity: 20/150 or better uncorrected, correctable to 20/20, each eye. Blood Pressure: 140/90 measured in sitting position. 3. Height between 58.5 and 76 inches.
Pilot Astronaut Candidate:
1. Bachelor's degree from an accredited instution in engineering, biological science, physical science, or mathematics. Degree must be followed by at least three years of related progressively responsible, professional experience. An advanced degree is desirable. Quality of academic preparation is important. 2. At least 1000 hours pilot-in-command time in jet aircraft. Flight test experience highly desirable. 3. Ability to pass a NASA class I space physical which is similar to a civilian or military class I flight physical and includes the following specific standards: Distant visual acuity: 20/50 or better uncorrected, correctable to 20/20, each eye. Blood pressure: 140/90 measured in sitting position. 4. Height between 64 and 76 inches.
Citizenship Requirements. Applicants for the Astronaut Candidate Program must be citizens of the United States. Note on Academic Requirements Applicants for the Astronaut Candidate Program must meet the basic education requirements for NASA engineering and scientific positions -- specifically: successful completion of standard professional curriculum in an accredited college or university leading to at least a bachelor's degree with major study in an appropriate field of engineering, biological science, physical science, or mathematics. The following degree fields, while related to engineering and the sciences, are not considered qualifying: -Degrees in technology(Engineering Technology, Aviation Technology, Medical Technology, etc.) -Degrees in Psychology(except for Clinical Psychology, Physiological Psychology, or Experimental Psychology which are qualifying) -Degrees in Nursing -Degrees in Social Sciences(Geography, Anthropology, Archaeology, etc.) -Degrees in Aviation, Aviation Management, or similar fields.
September 26, 2008Sean (California)How do I create a continuous electric arc? I was thinking something along the lines of running an electric current through a controlled stream of a highly conductive gas.This method is one of the most commonly used today. See, for example, this article.
September 25, 2008kali (goergia)are minerals compound or mixtureMinerals can be found in both forms. A compound is when the materials are chemically bonded (for example, salt is a compound of sodium and chlorine), while mixture is when the materials are intermingled but still keep their individual properties (granite is an example).
September 18, 2008PATRICIA (CALIFORNIA)IS THE ECLIPSE DANGER FOR HUMANSNo, eclipses, from the earth point of view, are just the partial or total apparent darkening of the sun when the moon comes between the sun and the Earth (solar eclipse), or the darkening of the moon when the Earth is placed between the Sun and the moon. The only related danger is to look directly to the Sun during eclipses without eyes protection.
September 17, 2008Sarah (Victoria)How do scientists discover their discoveries?After a lot of work and using the scientifc method.
September 17, 2008cya (india)if bubble gum sticks on our face why dosen't it sticks on our teeth?There are two main factors that differentiate one from the other, teeth are covered by a very polished, non-porous material and are always wet, the skin don't.
September 17, 2008cya (india)sir,what is the normal iq of a normal person? sir
Einstine's and sir newton's iq was more than a normal person?
The average IQ of the population as a whole is, by definition, 100. IQs range from 0 to above 200, and among children, to above 250. However, about 50% of the population have IQs between 89 and 111, and about 80% of the population have IQs ranging between 80 and 120, with 10% lying below 80, and 10% falling above 120. Adapted from What Does "IQ" Stand For, and What Does It Mean?.
September 16, 2008Molly (West Virginia)Can astronauts see asteroids, meteors or comets when they are on space missions?Although I haven't read about a specific case, I would expect astronauts to see them with more clarity because the lack of atmosphere at those heights.
September 16, 2008Devon (Ohio)A Scientist, examining a set of data, finds a single break in an otherwise strong trend or pattern in the data. Would the single break invalidate the trend or pattern? Why?No,a data point outside the main trend (known as an outlayer) could be due to many factors, like normal variability of the system, natural or artificial noise, etc. This is a extremely comun finding when working with "real" data. The use of error bars takes care of the variability.
September 15, 2008darryl (Illinois)Is the lake Geneva WI lake a crater?The second paragraph on this link describes the origin of the lake, and the rest that of the city of the same name.
September 12, 2008Min (Myanmar)What is mean Dwarfs?These are the meanings of dwarf from wordnet.princeton.edu: - a person who is markedly small - gnome: a legendary creature resembling a tiny old man; lives in the depths of the earth and guards buried treasure - shadow: make appear small by comparison; "This year's debt dwarfs that of last year" - a plant or animal that is atypically small - check the growth of; "the lack of sunlight dwarfed these pines" Currently is also used to identify the smallest (dwarf) planets in our solar system.
September 8, 2008Bears class (United Kingdom)How do rockets land?Rockets are not designed to land. Once all the combustible in them is used, they are released from the space ship and burn during re-entry in the atmosphere.
September 4, 2008priyanka (NEPAL/kathmandu)What we have to know at first to become a scientistThere are as many answers to your questions as scientists are, but I am sure we all agree in several common things -a love for learning, a big pasion for discovery, curiosity, math, and literal skills are basic factors over which a scientific carrer could be built.
September 1, 2008Amir (Pakistan)When a helicopter is placed on earth, its motion along with earth can be understood but when the helicopter is above earth (in air) why not it is displaced simply because of earth's rotation?
Which force binds helicopter to move along with earth when it is in air? (because gravity can only force it to come down not to move alongwith earth.)
The controlled motion of helicopters is a complex topic beyond the reach of this section. Let's only say that the combined, controlled motion of both rotors, including the pitch angle, is what keeps the helicopter in the right direction.
August 24, 2008Julia (Australia)How does Climate change relate to social justice and equity?You can find the answer to your question on Climate change, vulnerability and social justice, a paper published by Harvard scientists.
August 20, 2008Roy (North Carolina, USA)Is the oil that we are using up so quickly a form of insulation from the Earths molten core,Most of the oil is quite close to the surface, and I don't think it is enough, or well-distributed to be thought as a form of insulation.
August 20, 2008kav (trinidad)why is a flat tyre not considered to have zero atmospheres when flat?A flat tire still has air in the interior, only that it is not enough to use the tire as intended.
August 19, 2008brenda (nc)What is the most likely reason cars are designed with smooth surfacesI would say to minimize air friction.
July 2, 2008Darius (Baltimore)What is the hardness of the mineral cobalt (not cobaltite) on Mohs hardness scale?Cobalt has a hardness of 5.5 on the Mohs scale of mineral hardness.
July 2, 2008Keejhontai (u.s)were does the word metamorphic come from?The word metamorphic has Greek origins. Meta means change, and morphic means structure.
July 2, 2008emma (Michigan USA)What is an electrical surge that surge plugs protect against.An electrical surge is the short-lived, intense peak of electrical current that can reach our homes due to short-circuits, atmospheric electric discharges, etc. They could be quite damaging for common household appliances.
July 2, 2008Janice (San Antonio, Texas, USA)My nephew wants to know: If you throw a boomerang on the moon, will it come back to you? Or, just keep going straight?Bommerangs motion involves aerodynamic laws. It needs air to fly. See this website for more information.
June 30, 2008Timothy (Texas, US)How the moon is able to follow you when you are in a car ride?The average distance from Earth to moon is about 384 401 km. A normal car trip (for example, the distance between San Francisco and New York is 4125.91 km) is too short to be considered a change of position against the moon.
June 30, 2008Yu (Singapore)How many planets are there in the whole Universe?It is impossible to say. We don't have enough observational power to reach all corners of the universe.
June 27, 2008Haley (Massachusetts/USA)A person who studies and collects rocks for funA person who studies and collects rocks for fun is called a Rockhound. People that study rocks professionally are called Geologists.
June 25, 2008Saffron Could we ever live and survive on the moon?Due to the lack of breathable atmosphere it would be impossible without building an enclosed city or after a terra-forming process (the modification of the environment of a planet to make it more Earth-like and habitable by humans. Definition adapted from drernestmaurer.com).
June 20, 2008MarkWhat is the meaning of the tilde ~ ahead of the diameter and orbital period for Dysnomia, the moon of Eris in the Table of Moons?It is commonly used to identify approximate values.
June 19, 2008ROSE (NEW YORK)WHAT PROCESS DOES OUR EYES GO THROUGH IN ORDER FOR US TO SEE?This is a very complex topic, but in essence, our brain receives (and processes!) the retinal stimuli. The process in more details is as follows: the lens of the eye focuses light on the photoreceptive cells of the retina, which detect the photons of light and respond by producing neural impulses. These signals are processed by different parts of the brain. Adapted from (and more information) here.
June 18, 2008Savannah (virginia)How would you create a teaching poster on earth's atmosphere?I would include the different layers, their height, composition and temperature profile, and the natural and artificial events in each one. We have several web pages on this topic, for example visit The Earth's Atmosphere and the links therein.
June 18, 2008Savannah (virginia)How would you create a teaching poster on eart's atmosphere?I would include the different layers, their height, composition and temperature profile, and the natural and artificial events in each one.
June 17, 2008omar (pakistan)Some books describe that Earth s rotation time is 23 hours, 56minutes 4 seconds.So why our day is of 24 hours and where these remaining 3 min and 56 seconds go?All this topic is related with the fact that natural processes (like seasons and astronomical events) do not repeat at an exact number of full days. So calendars have to be adjusted every certain number of years to account for this difference. This is the reason foe Leap Years, when we add an extra day to the calendar.
June 16, 2008esraa (cairo, egypt)how can I design a simple telescope to watch stars from my home ?
PLEASE ANSWER MY QUESTION I love stars very much
In this page there are simple and clear instructions for telescope designs.
May 16, 2008Chinika (Trinidad)Did Albert Einstein work on the atom?Einstein did not work (at least in any significant way) on the atom. Although Einstein did promote the research toward the atomic bomb, he did just a minimal amount of related work. He was more a scientists with very novel ideas.
April 28, 2008Larry (Missouri/USA)If I start a trip up the Prime Meridian at 6AM and my brother travels up the 180* meridian starting at 6PM, and we both reach the geographic north pole exactly 12 hrs later, who will have the correct time on his watch, 6PM or 6AM?The difference between the length of the night and the length of the day gets bigger the further away you get from the equator in the direction of a pole. A summer's day (a winter's night) at the poles lasts 24 hours without any night (without any day). Thus, the time difference between the watches is not important.
April 22, 2008Ben (Missouri/USA)What advice did Bohr give to the United States government after the first atomic bomb test in 1945?After the first use of an atomic bomb at the end of the II World War, Bohr and other scientists played an important role in the preservation of the peace, and in the struggle against proliferation of nuclear weapons. However, before the end of the war Bohr saw favorable aspects in the spread of nuclear weapons as it would put an end to war-"completely changing all future conditions of warfare" and making it impossible to win.
April 10, 2008Katharine (CA, USA)I'm doing a project on cone cells. I was wondering if you could give me some information on them like:
its functions
any specialized structures
how it is different from other types of cells
why it is shaped the way it is
what size the cell is
how long it lives
where it is found
any other interesting facts about it
I need to write an essay with at least 4 paragraphs and 5 sentences in each paragraph. I also need a picture of it with all its organelles because i need to make a model. Any suggestions on model items I can use? I can use anything as long as it is not perishable.
The cone cell is a type of light-sensitive nerve cell found in the retinas of most vertebrates. Cones are concerned with the discrimination of color and of fine detail. In humans, they contains three types of of photosensitive pigment, compared to just one in rod cells. They also have a much faster response to light than do rods, so that they can perceive more rapid changes in stimuli. Adapted (and more information, including a diagram) from The Internet Encyclopedia of Science.
April 9, 2008 Why not use the Earth's core or tap into an active or dormant volcano as a clean and unlimited power source.There are new techniques, or old ones adapted to the current conditions, that use heat from underground. For example, geothermal heat pumps use natural thermal energy stored just below the earth's surface to provide space heating during the winter and cooling in the summer.
April 2, 2008Adriana (new mexico)on your list of ecosystems Mountains is not mentioned, why? my teacher says it is one of the ecosystems.There are different definitions (and listings) for "ecosystem". Examples of ecosystem include: aquatic ecosystem, chaparral, coral reef, desert, human ecosystem, large marine ecosystem, marine ecosystem, rainforest, savanna, subsurface, lithoautotrophic, microbial ecosystem, taiga, tundra, urban ecosystem, etc.
April 1, 2008mackenzie (kentucky)what is one fact about evolution?One of the strongest facts about evolution is the long Fossil Record collected by scientists that document changes in past life on Earth.
March 31, 2008Anthony (North Carolina)Something has been puzzling me. It is my understanding that without the electron, there can simply be no emission spectrum because there would be nothing to excite. It seems then to me that viewing protons or neutrons would be an impossibility. Is this correct?. Also, I was wondering if anyone has ever tried to excite such particles in order to test that there is no emission. One last question, can there be a star, i guess an ionized star that has no electrons and is therefore possibly invincible to detection by meens of light? Thank you for any assistance with any one of these questions.The problem with this hypothetical question is that without the electron the atoms would be always in an altered state, which would be against our understanding of the universe. The continuous interchange of particles is part of the dynamic of the universe.
March 25, 2008jocelyn (Malacca,Malaysia)and when they return to EIf a group of astronomy traveling in universe several years,and when they return to Earth,will they also bring back other bateria that are dangeous to human kinds?NASA, and other space agencies in the world, have implemented strict rules for astronauts returning to earth, and the samples from other planets or moons they could bring back. See this article foe a little piece of history on this topic.
March 21, 2008Rosalind (Maryland/USA)Why is it that models of the solar system show the sun to be the size of the planets when the sun is the largest mass in the solar system by far! Why don't models accurate show the planets as marbles or basketballs to the sun? Am I corrrect?You are right in all accounts. A representation with a realistic proportion of our solar system will be almost impossible to do in a normal media (paper or screen), there won't be room enough to represent all the planets!
March 21, 2008Gerence (UK)How much Heat Energy is released from the Oxidation of (O1)Carbon and (2)Hydrogen?The oxidation of carbon to CO2 releases a significant amount of energy (heat). For example, if a gram molecule of glucose is simulated under aerobic conditions, up to 674 kilogram calories (kcal) of heat may be released. Adapted from urbangardencenter.com
March 20, 2008Tim (Australia)Can you insert a tiny projector inside a thin cylinder and activate 3D holographic images to be projected in mid-air; is technology up to this?For a discussion on the possibilities of holographic projections, visit here.
March 18, 2008Kurt (North Carolina, US)light bouncing back from an object to its source is what?Not sure I understood your question, but, in general, it is called reflection when the light "bounces" back from an object.
March 17, 2008Lindsey (Indiana)What are the general traits for each of the four rock types?Geologists classify rocks in three groups, according to the maian processes that formed them. Sedimentary rocks are formed via weathering and transport of existing rocks, and then deposition, cementation, and compaction into a sedimentary rock. Igneous rocks are formed by cooling and crystallization of molten rock. Metamorphic rocks are formed when heat or pressure are applied to other rocks. More information on rocks and rock cycle in our web page What is the Rock Cycle?.
March 17, 2008Brittany (Atkison)what is luster?Luster, in general, can be define as the appearance of the reflection of light from a surface.
March 14, 2008desiree (memphis tn)who was the twevle gods of greek mythologeThese twelve gods are called the Olympians. they are Zeus, Poseidon, Hades, Hestia, Hera, Ares, Athena, Apollo, Aphrodite, Hermes, Artemis, and Hephaestus.
March 5, 2008rebecca (england)what causes shadows in the first placeShadows are merely caused by the interposition of an opaque object in the path of light rays.
February 25, 2008Alice (India)I WANT TO BECOME AN ASTRONOMER SIR.AND MY FIGURE ISN'T GOOD.BUT I WANT TO BECOME IT.I KNOW THERE IS SOMETHING WHICH I WANT TO DISCOVER.AND IS IT TRUE THAT WHEN ANY ONE GOES THROUGH TIME MACHINE HE WILL GO TO AN ENTIRELY NEW UNIVERSE ?No "time machine" has been ever created!
February 25, 2008Emma (England)I always wondered if the universe is expanding (as it is said to be) then what is it expanding in to. Our strongest telescopes can only see as far back in to the universe as the radiation guessed to be left over rubbish from the big bang. I looked through the questions already asked and found this question: The universe is everything that exists. So, if the universe does expand forever, what will it expand into? The answer was: What the universe expands into depends on whether it is infinite or not. If it is infinite, it is not expanding into anything, since it has no edge. If it is finite, this means that space is curved back on itself like a balloon. This would mean that space had no edge, only a smooth surface. In this case, the universe could not be expanding into anything like space-like. As galaxies get farther away, space stretches, but nothing is displaced.
My point is that nothing cannot exist, nothing is impossible, there much be something. If it bends back on itself in an oval like shape there must be something behind this oval. I strongly disbelieve that there is nothing. If it is expanding there has to be room for it to expand in to otherwise it would not expand. Take a balloon inside a box. Blow the balloon and when it gets too big for the box it will not expand anymore, there is no room for it. If there is nothing stopping the expansion what is it expanding in to. If it is bending back on itself what is outside of this shape. Could that be a parallel universe?
The answer to your question escapes the scope of this section. For a good discussion of this topic, visit here.
February 13, 2008tikeyah (illions)how does the concentration of salt water affects how well it freezesFor higher concentration of salt, the freezing point of water will be greater. See this webpage for a detailed explanation.
February 12, 2008Debbie (Michigan)I found a interesting rock in Grand Traverse Bay that looks like a foot print is embedded in it, the foot print looks somewhat human, is about 11-12 inches long, it gets really wide at the toe end 5-6 inches. Where can I take this rock to have it check out by someone reliable? The University of Michigan Museum of Anthropology (UMMA) is a good and nearby location for you.
February 11, 2008Ian (Ontario)What is anything divided by zero. If it is not possible then what are some of the educated guesses out there?Division by zero is an operation for which there is not an answer, it is undefined.
February 11, 2008Emily (P.A)What is the meaning of a sediment?Sediment is any particulate matter that is deposited as a layer of solid particles on the bed or bottom of a body of water.
February 8, 2008Radha (India)I have heard that our earth is about to finish in coming 65 years. is it true?There is not any factual basis for such a claim.
February 7, 2008William (Illinois)Why do soda pop go flat?Because the carbon dioxide, which keeps the soda carbonated, escapes when the bottle or can is open.
February 7, 2008harris (florida)why does a soda bubble when you shake itThe carbon dioxide gas, which keeps the soda carbonated, is usually collected at the top of the liquid, in the space between the cap and the surface of the soda. When the bottle is open, the pressure inside the bottle change and the gas suffers a sudden expansion, escaping with a hiss from the bottle. When the bottle is shaken, the gas is mixed with the liquid, and if the bottle is opened immediately, the gas expands and forces, during this process, the liquid out of the bottle.
February 6, 2008Anne (UK)What kind of enzyme is used in washing powders and how does it work to get your washing clean, in terms of the active site and substrate?
What are the adavantages and disadvantages of using a biological washing powder rather than a non-bio alternative??
What links do biological washing powders have to dermatitis?
Your questions escape the reach of this section, but you will find a good deal of information on this topic here.
January 31, 2008norman (ontario)my question is did you guys find out if there life on titain one of saterns moonsbecause i hered of that prob u sent there?
and if u guys need some one to go to mars in 2030 call me!
So far, the efforts to find extraterrestrial life have not been successful.
January 30, 2008Sylvia (Alaska)what is cleavage fracturePlease, visit this page.
December 27, 2007Gabie (Colorado)Why does America belive in climate models and how does it help our kind?A climate model uses hundreds of mathematical equations to describe processes that happen on our planet, processes like wind, ocean currents, and plant growth. Math is also used to describe how Earth processes are related to each other. For instance, how wind patterns affect the transport of storms from one area to another, how ocean currents affect the amount of heat in the atmosphere, and how plant growth affects the amount of carbon dioxide in the atmosphere. All these things, described with all this math, help scientists understand how Earth's climate works and how climate is changing. More information in our web page How Climate Models Work.
December 27, 2007Elena (USA,Colorado)Why does the scale show our mass in pounds if,in fact,it does measure our weight,which should be in newtons?You are right. I believe it is only a matter of convenience or what we are use to.
December 26, 2007Carla (Florida)How many satellites are in space?
How many are active and inactive?
How far are they from earth?
How do you put them in space?
An exact answer to your question is hard to come with. It is accepted that more than 2200 tons of manmade junk are in the environment near the earth. The main "space polluters" are Rusian and USA, as well as the main users of active satellites (close to 4 thousands in each case).
December 26, 2007jamyra how does plasma produce picturesA detailed answer to your question here.
December 26, 2007mohammed (uk england)what is the difference between the baby moon and the sightings of the moon?The definition of babymoon is: Babymoon, a planned period of calm spent together by a just-born baby and its parents; occasionally, time spent by parents without their baby; (hence) a vacation or holiday taken by a pregnant woman and her partner. This is not related to our Moon in any way.
December 21, 2007Shailee (Ontario, Canada)How can I communicate with a scientist, an astronaut or an astronomer?There are many space scientist working in universities all around the globe. For example, there is an Institute for Space Studies in the University of Toronto, Ontario (where you live).
December 21, 2007corbie (canada)is it true that all minerals can form crystalsMinerals grow in specific shapes, and usually crystallize into one of six crystal systems: Isometric -- Example: pyrite. Tetragonal -- Example: zircon. Hexagonal -- Example: beryl. Orthorhombic -- Example: topaz. Monoclinic -- Example: gypsum. Triclinic -- Example: feldspar.
December 21, 2007Carri (KY, USA)Can You See Lighting With The Human Eye?Yes, the electric discharge of lighting is visible.
December 19, 2007michael (texas)What is the definition of physical and chemial changeChemical Change: A rearrangement of the atomic array of matter which results in a change of physical properties. Physical Change: A change in matter which does not alter the chemical properties of the matter.
December 18, 2007rix (phillipines)how do satellites stay at their orbits?The determination of satellite's orbits involves very complex calculations. If the satellite is supossed to stay in orbit around earth, then it would need a particular speed, but if the stellite will leave our planet, then it would need a different, and higher speed (these are called escape speed). To keep a satellite in orbit, gravity and the momentum of the satellite have to be balanced.
December 18, 2007robernae (ky usa)in order to watch tv does there have to be a sattellite in spaceAlmost everywhere there are local stations broadcasting, some of them even in high resolution, and you only need a normal antenna to receive their signal.
December 17, 2007kelsey (PA)What is geology?Geology is the science and study of the solid matter of the Earth, its composition, structure, physical properties, history and the processes that shape it.
October 16, 2007alicia (ga)where can i find the roman god "fortuna" at?This page will give information on this topic.
October 15, 2007mary (england)what is the connection between the size of a planet & it's densityDensity is defines as mass divided by volumen. If the mass of an object is kept constant, the density of the object will decrease when the object increases its size (volume).
September 24, 2007reneehow have humans used technology to explore space?The exploration of space has been based on the technological and scientific advances of humanity. The development of the powerful engines, the advances in radio communication, radar technologies, optical technologies for telescopes, understanding of the laws of the movement, the application of Physics, Math, etc. are only few examples of the use of science and technology in the exploration of space.
September 24, 2007karinawhat is a criteria?Criteria are a set of standards by which something or someone can be judged so as to enable a decision to be made as to the identity or classification of any subject. Extracted from UNDP.
September 24, 2007RANGARAJAN (INDIA)If the other side of ocean can also be an ocean,then how come gravitational pull work on both the oceans which are diametrically opposite and still holds the water towards earth?Gravity is a common force first studied by Newton. He expressed his Law of Universal Gravitation as "Each particle of matter attracts every other particle with a force directly proportional to the product of their masses and inversely proportional to the square of the distance between them." The mathematical formula for the force of gravity is: Fg = (G * m1 * m2) / (r2) where G is the gravitational constant (a very small value (6.67 x 10-11 N m2 kg-2 = 0.000 000 000 066 7 N m2 kg-2 )), m1 and m2 are the masses of the two objects, and r is the distance between the two masses. The gravity force calculated by this expression acts between the centers of masses of the bodies, so it is not important where the oceans are, everything on our planet is under the action of the gravitational force that it exerts.
September 24, 2007RANGARAJAN (INDIA)if our earth is like a ball,we live on the top surface of the ball.If an aeroplane flys continously in eastern direction,then how it reaches the western side as there is no change in the flying direction of the planeNorth, South, East and West are defined as a function of the observe position. So, there is no a way to "reach" them, they move with you.
September 18, 2007Michele I have been getting emails from people say that on Auugust 27, 2007 at 12 :30 am mars will be so close that it will look like earth has 2 moons. Is this true?This story is not true. It is a myth that has been floating around the Internet for a few years. Mars will never look any larger than a bright star to the naked eye. The next close approach between Earth and Mars will be in mid-December of 2007; the planets will pass within about 88 million km (55 million miles) of each other. In August 2003 the two planets were closer together than they had been for a very long time. See the following pages for more: Headline Universe, Mars Orbit. We try to highlight at least some of the major ongoing astronomical events; but unfortunately do not have enough staff to cover all of them. (Thanks to Randy).
September 18, 2007abdul (india)can electricity is supplied without a wireYes, and a good example are RFID (radio-frequency identification) tags, used for the purpose of identification using radiowaves. An interesting application is described here.
September 12, 2007aadya (India)in school, we put a portable music system on the floor, where some water had spilled and then plugged it into an electrical socket to play it. it played for a short while and suddenly there was smoke coming out of the music system and it blew up! the mini-circuit-breaker was still 'on' but the other appliances like fans etc also went off. what happened here? could you please explain??Just in few words, electricity and water is always a dangerous mix!!! Water has a low electrical resistance, and the electricity always follows the lower resistance path.
August 17, 2007Nishi (United Arab Emirates)what is Kyoto ProtocolFrom December 1 through 11, 1997, more than 160 nations met in Kyoto, Japan, to negotiate binding limitations on greenhouse gases for the developed nations, pursuant to the objectives of the Framework Convention on Climate Change of 1992. The outcome of the meeting was the Kyoto Protocol, in which the developed nations agreed to limit their greenhouse gas emissions, relative to the levels emitted in 1990. The United States agreed to reduce emissions from 1990 levels by 7 percent during the period 2008 to 2012. The full text of the protocol in several languages can be found in this link.
August 15, 2007Dylan (auckland)how many km is a light yearA light year is the distance that the light (or any electromagnetic signal) covers in the vacuum in one terrestrial year. 1 light year equals 9.4605284 × 1012 kilometers, this is 9 460 528 400 000 kilometers.
August 14, 2007NAINA (LUCKNOW)WHAT IS PERIMETER AND AREA?Perimeter is the measure of the distance around the outside of a shape.area is the amount of space in a flat surface measured in square units.
August 14, 2007NAINA (LUCKNOW)WHAT IS STONE AGEThe Stone Age is part of the history of the world that comprises the first extensive use of (rudimentary) technology in human evolution. The Stone Age is usually divided into three separate periods--Paleolithic Period, Mesolithic Period, and Neolithic Period--based on the degree of sophistication in the fashioning and use of tools.
July 26, 2007sam (birmingham)what was the name of the first clock ever madeDevices to keep track of the time have been used by humans since immemorial times. From sundials and sand clocks (a.k.a. hourglass) to atomic clocks, humans have always kept track of time.
July 25, 2007srividhya (india)Who was one who found out science?The origins of science are not clearly established. While empiricism has been described since Antiquity (Aristotle), and the scientific method has been employed since the Middle Ages (Ibn al-Haytham), the modern scientific method was not fully developed until the Scientific Revolution, during the 16th and early 17th centuries. More on this topic here.
July 25, 2007Bertus (South Africa)Wat exactly are CTCs?Your question is really open. Here are many meanings for CTC. Now, if you were asking about CFCs, more related to our web site, they are chlorofluorocarbons, a family of chemical compounds that have negative effects on the environment such as ozone depletion.
July 23, 2007Ndeche (Nigeria)how do i construct a small frequency fm radio stationYour question escapes our content, but I can direct you to web sites that offer the information you are looking for. If you want to build a FM receiver, please go to this page. If you want to construct a small transmiter, visit this
July 23, 2007avaan (india)why are carbon-dioxide emissions higher in winter?Weather is a very important factor, although not the only one, in the emissions of carbon dioxide. During winter, there is an increase in the consumption of heating fuels, and, of course, in the emissions of energy-related carbon dioxide.
July 23, 2007Millard Paul (New York)What happened to Uranos when cascrated by Cronus?Neptune was the name that ancient Romans gave to the Greek god of the sea and earthquakes, Poseidon. He was the brother of Jupiter (Zeus) and of Pluto (Hades). After the defeat of their father Saturn (Cronos), the three brothers divided the world in three parts to be ruled by one of the three brothers. Jupiter took the sky, Neptune the sea and Pluto the underworld.
July 23, 2007Adolfo (Texas/US)the mayan calender claims there will be an astronaumical occurance in the year 2012, what could it be if it marks the end of their era?This article should answer your question. Let me quote the last three paragraphs: "But scholars doubt the ancient Maya extrapolated great meaning from anticipating the alignment — if they were even aware of what the configuration would be. Astronomers generally agree that "it would be impossible the Maya themselves would have known that," says Susan Milbrath, a Maya archaeoastronomer and a curator at the Florida Museum of Natural History. What's more, she says, "we have no record or knowledge that they would think the world would come to an end at that point." University of Florida anthropologist Susan Gillespie says the 2012 phenomenon comes "from media and from other people making use of the Maya past to fulfill agendas that are really their own."
July 21, 2007elisa (California)why is neptune called like that.In the Roman mythology, Neptune was the name that ancient Romans gave to the Greek god of the sea and earthquakes, Poseidon. He was the brother of Jupiter (Zeus) and of Pluto (Hades). After the defeat of their father Saturn (Cronos), the three brothers divided the world in three parts to be ruled by one of the three brothers. Jupiter took the sky, Neptune the sea and Pluto the underworld.
July 20, 2007Gwendolyn (New York, USA)What is the molarity of salts in sea water?A detail answer to your question appears here.
July 19, 2007siddharth (uttar pradesh,india)how can a space shuttle come back to earth from moon without additional fuel as the engines are dropped off during the journey?Actually, they Space Shuttle engines are not used for lift up. The power at this time is provided by external fuel rockets that propel the shuttles to their escape velocity. The on-board engines are use for maneuvers in space, including the orientation, acceleration (and braking!) when returning to Earth.
July 18, 2007Stefan (Romania)Is it possible to design a metal based alloy that would withstand the ionosphere transition ? What are the characteristics of such an alloy melting temp/ electrical conductivity , electrical resistance,?
Which is the maximum temperature that would be applied upon an object rising vertically towards space and what are the requirements for such an object to achieve reentry in one piece
Without getting in to the details of your question, the answer is yes. All shuttles and rockets go through the ionosphere without major damage.
July 18, 2007LaTashia (GA)Where can I find a useful URL guided discovery activity on Earth and Space and the Processes of Life?You have come to the right place to find answers. Our website covers many topics related to Earth, Space and Life, including hands-on activities that teachers and parents can use. See, for example, the list of activities in our web page Space Weather Hands-on Activities for Classroom & Home.
July 18, 2007jessica (michigan)what did the ancient roman people believe causes volcanic eruptions?The word 'volcano' comes from the little island of Vulcano in the Mediterranean Sea off Sicily. Centuries ago, the people living in this area believed that Vulcano was the chimney of the forge of Vulcan - the blacksmith of the Roman gods. They thought that the hot lava fragments and clouds of dust erupting form Vulcano came from Vulcan's forge as he beat out thunderbolts for Jupiter, king of the gods, and weapons for Mars, the god of war. In Roman mythology, Vulcan, the god of fire, was said to have made tools and weapons for the other gods in his workshop at Olympus. Throughout history, volcanoes have frequently been identified with Vulcan and other mythological figures. Extracted from Volcanoes and Mythology.
July 10, 2007Mikhail (Australia)what was Posiden and Medusa's child called?Upon her death, the seeds of the union of Poseidon & Medusa germinated into the young colt named Pegasus and the giant named Chrysoar. More information on this topic here.
July 9, 2007kritii (india)what are man made satellites?
what are natural satellites?
Natural satellites are bodies that orbit the planets, like the moons. Man-made satellites are artificial object created for us to explore the solar system. They are not natural objects.
July 9, 2007Jorden (Ca.)Why are planets named after Roman gods?In Roman mythology: Mercury was the messenger-god of Jupiter, and was the god of games, of business, and of story telling. Venus was the goddess of love and beauty. To her perfect figure and pure features she added an innocent manner. Mars was identified with the Roman god of wars. He always acted like a strong warrior and a strong desire for violence. Jupiter was the king of heaven and Earth and of all the Olympian gods. He was also known as the god of justice. In Greek mythology, Cronus was the son of Uranus and Gaea. The Romans adopted Cronus as the god Saturn. Uranus stands for the starry sky in the evening. In the creation myth of the Greeks, Uranus suddenly came out of the Earth (shown as the goddess Gaea). Gaea herself suddenly came into being out of Chaos, which came before all things. Neptune was the name that ancient Romans gave to the Greek god of the sea and earthquakes, Poseidon.
July 5, 2007Tom (United states)how long does it take an rocket to travel the whole solar systemTo calculate the time we need to know the velocity of the rocket, if it is constant or not, and if the rocket is traveling in a straigh path.
June 28, 2007shaun (texes)what is a meaterThe formal and official definition of meter is: The meter is the length of the path travelled by light in vacuum during a time interval of 1/299 792 458 of a second. Visit NIST for more information.
June 28, 2007liz (ca)how do scientist know what the structure of the earth's interrior isThere are several methods to study the interior of our planet. For example, seismic waves offered the first glimpse in to our planet's core. Here, in the section Exploring the Earth's Core, you can find an interesting and clear description of how the earth's core was discovered.
June 28, 2007grace (england)how do you measure the diameter of the earth?The diameter of our planet was measured by Eratosthenes in 230 B.C. Follow this activity from the University of Texas, to obtain the diameter of our planet by yourself (and a friend).
June 28, 2007Stephen (Oregon United States)The process to find the age of a rock using relative dating is?Looks like you answered yourself: Relative dating tells scientists if a rock layer is "older" or "younger" than another. This would also mean that fossils found in the deepest layer of rocks in an area would represent the oldest forms of life in that particular rock formation. In reading earth history, these layers would be "read" from bottom to top or oldest to most recent. If certain fossils are typically found only in a particular rock unit and are found in many places worldwide, they may be useful as index or guide fossils in determining the age of undated strata.
May 23, 2007brooke ()how do you know the planets are in spacePlanets, due to their different behavior on the sky, are easily spotted by the unaided human eye. Many modern techniques are available now to study the planets in details, from space missions to spectra analysis, we now know their density and composition, size, distance from the Sun, etc. For a good amount of information about the planets, visit our web page Our Solar System.
May 21, 2007jackson (birmingham)Is the law of attraction the same as the law of gravity if so do gravity and attraction mean the same thing i.e. Newton's Law of Gravity/Attraction?Actually no, Newton's Law of Gravity is a universal law, based on scientific observations and method. The law of attraction refers to people experiencing the corresponding manifestations of their predominant thoughts, feelings, words, and actions. Hardly a scientific conclusion.
May 10, 2007Jerky (Indonesia)What is ScienceScience is the term that refers to: 1. the scientific method – a process for evaluating empirical knowledge 2. the organized body of knowledge gained by this process.
May 10, 2007abu (Canada)What is the difference between emitted and reflected light?Emitted light, like the one coming from the Sun, is produced by the body, while reflected light is produced by a different body, and just bounced by the body being observed.
May 10, 2007jarod (ohio)What should you do during a thunderstorm?Adapted from the FEMA website: If you cannot find a shelter, get in to the "lightning safety position," squat down near the ground with your heels touching and put your hands over your ears. Do not go under a tree for shelter because if you do the lightning could hit the tree and travel under the ground or "splash" from the tree and hit you.
May 10, 2007Christopher (NY )What shapes do crystals form into?The shapes of mineral crystals are cubic, tetragonal, orthorhombic, hexagonal, monoclinic, and triclinic. To see these shapes, visit our web page Crystal Shapes.
May 5, 2007lalalala (PN, USA)What is the theme of the myth about Persephone and how she was kidnapped by Hades?Persephone was the Greek goddess of the Underworld and the wife of Hades. Romans identified Persephone with Proserpina. Hades fell in love with her at her first sight. He decided at once to carry her off to his kingdom. Demeter was the mother of Persephone. She wandered the world in search of her daughter. When she learned her daughter's fate, she pleaded with Zeus to obtain the release of Persephone. Unfortunately, Persephone had eaten some seeds of a fruit during her stay in the lower world, and this stopped her from leaving forever. After much crying and sadness, Persephone was allowed to spend half of each year on Earth with her mother, and the other half in the underworld. She became an allegory of death and rebirth of seasons. From our web page Persephone.
April 19, 2007Nichole I want to know what about SpectroscopeSpectrometers are used in spectroscopic analysis to identify materials. Spectroscopes are used often in astronomy and some branches of chemistry. This NASA-activity will help you to build a basic spectroscope, and in doing so, to understand the principles of refraction.
April 4, 2007Diana (Guam)How do humans affect our local ecosystem?Humans affect the ecosystems in many ways. From the abuse of resources, to the pollution generated, there has not been until recently a clear conscience of the damage inflicted to our planet. The use of fossil combustibles is one of the most clear (and critical) examples of the ways in which we affect the environment.
March 31, 2007Shelby (south dakota)Who was ulysses? I know that he was a roman hero and that the sun was named after him but who is he?The legendary Greek hero, Odysseus was the king of Ithaca, a small island in the Ionian sea, where he lived with his wife Penelope. He was known to Romans as Ulysses. After fighting the war against the city of Troy with the Greeks, he started his journey home. His sailing journey was obstructed by the sea god Poseidon. During his journey he had many incredible adventures, but his renowned intelligence helped him to survive the numerous difficulties, although his crew wasn't so lucky. More information in our web page Ulysses.
March 30, 2007stacie (california)how long have people studied constellations?Constellations are formed of bright stars which appear close to each other on the sky, but are really far apart in space. The shapes you see all depend on your point of view. Many societies saw patterns among the stars with gods and goddesses or stories from their culture.
March 30, 2007David (NewZealand)Is it possible to make a continum with an electric motor driving a generatorNo, there will be always some energy lost from Joule effect, dissipation, impedance mismatch, etc.
March 30, 2007David (Arkansas USA)If one was to put a microphone in space and recorded it what kind of sounds if any would it pick up??There are many sounds in space, for a small collection of actual sounds visit the web page Select Sounds of Space.
March 30, 2007sangini (india)what is the capacity of a human brain?I don't believe there is an accurate estimate of the human brain capacity, but an interesting discussion on this topic can be found at the web page How Many Bytes in Human Memory? .
March 28, 2007Maria (New York)What does a Kestral bird look like?You will find the information requested here.
February 25, 2007Bethany (PA/ United States of America)What were the twelve difficult tasks Hercules had to do when Hera casted the spell on him?Hercules' twelve tasks are know as: Labor 1: The Nemean Lion Labor 2: The Lernean Hydra Labor 3: The Hind of Ceryneia Labor 4: The Erymanthean Boar Labor 5: The Augean Stables Labor 6: The Stymphalian Birds Labor 7: The Cretan Bull Labor 8: The Horses of Diomedes Labor 9: The Belt of Hippolyte Labor 10: Geryon's Cattle Labor 11: The Apples of the Hesperides Labor 12: Cerberus For a good description of them, visit The Labors of Hercules.
February 13, 2007Chrissy (NH)About how many blood cells are there in an average persons body?An average adult person body has about 1 to 1-1/2 gallons (4-6 liters) of blood.
February 13, 2007Mary ( Canada)Where did the gigantic stones at Carnac, France, originate?The Carnac stones are an exceptionally dense collection of megalithic sites around the French village of Carnac, in Brittany, consisting of alignments, dolmens, tumuluses and single menhirs. The more than 3,000 prehistoric standing stones were hewn from local rock and erected by the pre-Celtic people of Brittany, and are the largest such collection in the world. Extracted from The Carnac Stones.
February 6, 2007melissa (Fl)Tell me all about me Greek God Persephonein to do a report on he Iam in 9th grade....thanks for your time,
Melissa Hunley
Persephone was the Greek goddess of the Underworld and the wife of Hades. Romans identified Persephone with Proserpina. Hades fell in love with her at her first sight. He decided at once to carry her off to his kingdom. More information about this myth can be found at our web page Persephone.
January 14, 2007Joy (Philippines)What is mythology?Mythology is the study of myths: stories of a particular culture that it believes to be true and that feature a specific religious or belief system. For more information visit the web page Mythology.
January 3, 2007Poonkodi (Tamil Nadu/India)How the myths are related to science fiction and modern science?To answer your question would require more space that the allowed, but you can find lots of related information in our web page Mythology.
January 3, 2007Vanamali (India)What is meant by Fourier Series? What is its importance?The Fourier series is a useful mathematical tool for analyzing periodic functions by decomposing it into a weighted sum of sinusoidal component functions (called normal Fourier modes).
December 27, 2006bob (florida)what does metamorphose meanDepending on the context metamorphosis can have different meanings: - the marked and rapid transformation of a larva into an adult that occurs in some animals - transfiguration: a striking change in appearance or character or circumstances; "the metamorphosis of the old house into something new and exciting".
December 27, 2006SHEA (SOUTH CAROLINA)WHAT DOES THE WORD MESO- MEAN?Meso is a Greek prefix meaning middle or mid. Is used with Latin, latinized, or Greek words to indicate the middle (often second) part of a structure.
December 9, 2006matt (california)what is paleoclimatologyPaleoclimatology is the study of climate change taken on the scale of the entire history of the Earth.
December 6, 2006Rita (Chia)What are the features of the Roman myths?This is an extensive topic to be discussed in a short answer. We have developed several web pages about this topic. They can be accessed at this page.
November 20, 2006David (Jamaica)What is a Acid RainCloud or rain droplets containing pollutants, particularly sulfur oxides and nitrogen oxides, products of burning coal and other fuels and from certain industrial processes.
October 19, 2006siedah (dominica)what are the names of the early civilizations
Early civilizations were extended all over the planet. Just a few of the best known are:

Europe / Middle East: Vikings, Celts, Phoenicians, Balkans, Minoans, Myceneans, Troy, Byzantine, Aegean, Etruria, Spartan, Macedonia, Constantinople
Africa: Nile Valley, Egypt, Kush, Ethiopia
Asia: Akkadians, Mesopotamia, Sumerians, Assyrians, Babylonians, Persian Empire, Maccabees, Parthians, Phoenicians, Ottoman Empire, Aryan, Gupta, Khmer, Huns, Mongol, Chinese Dynasties
Australia & Oceania: Maori, Polynesia, Australian Aborigines
America: Anasazi, Mayan, Olmec, Aztec, Toltec, Incas, Moche, Ancient Americans
December 14, 2000LC (New York, USA)I am obliged to do a science fair project in less than one week and it is to be on reactions in earth science. I am very baffled as to what to do.This page might just help!
October 9, 2000Scott (Florida, USA)Why does a circle have 360 degrees?To the Egyptians we owe the invention of the 24 hour day and the 365 day year. But, it is to the Babylonians that we owe the designation that a circle has 360 degrees. The Babylonian number system was based on sixty. In fact, you could safely say that it was their favorite number!

The smallest unit of length was the finger. A cubit was 30 fingers, the cord was 120 cubits or 3,600 fingers. And a league (about 7 miles) was 180 cords.

The smallest unit of weight was the grain. A shekel was 180 grains and a talent was 3,600 shekels.

The calendar was 360 days and the circle had 360 degrees. All measurements dependent on the number sixty...and so it was decided about 4,000 years ago when the Babylonians were a thriving civilization that the circle would have 360 degrees in it!

November 4, 1999Patrick (Wisconsin, USA)Has a hat with a portable fan to cool you down(placed in front of you), walkman to listen to music, soda cup holders with long straws so you could drink it(on the side of the top), and a flashlight on top to see in the dark all combined into one hat ever been invented?Not as far as I know! And I did a search of U.S. patents, and nothing like that came up. So, invent away!
May 19, 1999Kat (Australia)Can you please give me 2-3 examples of how we use Pythagoras' Theorem in every day life?Well, if you wanted to know the height of a building, so a plane doesn't hit it, you would use the Pythagorus'theorem. You can step back from the building like 50 ft. If you approximate the angle between the top of the building and where you are standing, you can calculate the height of the building! A simpler example would be during a basketball game. You know that the hoop is ten feet tall, and you know you are standing 10 feet from the base of the basket. You can use the theorem to find the distance between the basket and you!
May 18, 1999Clint (Ohio, USA)How do filmakers shoot weightless scenes? Like Apollo 13 or Armageddon.For Apollo 13 filmakers used the vomit comet, a NASA turbojet that simulates weightlessness for brief periods through a series of arc manuevers; Armageddon made use of NASA's neutral buoyancy tank; and both m ovies relied heavily on special effects, often created using computers.
May 18, 1999Alyssa (USA)What is the difference between Astronomy and astrology?Astronomy is the science (study) of stars and other objects outside of Earth's atmosphere. Astrology, on the other hand, is the belief that the movements and patterns of stars and planets effect or control peopl es' lives. See our mythology section to learn about cultural connections between astronomy and astrology.
May 11, 1999Patrick (USA)There is a sculpture of Perseus looking at the head of Medusa. Is this a joke((ie: the sculptor carved out of stone a man turned to stone by looking at the gorgon?) I thought it was funny, but I'm wondering if I'm the only one who noticed this. I would appreciate any response. Thanks!There is no way to know for sure, but the sculptor may have had humor on his mind when creating this piece. According to the story, Perseus looked at Medusa's reflection in his shield and killed her while she was sleeping. At the time, Perseus hid the head in a kibisis, which is a type of wallet.Pegasus was born from Medusa's body. Even after her death, the head of Medusa still turned anyone to stone that looked at it. The sculptor was either being funny or showing how powerful Perseus was because he could overpower this famous villain.
May 10, 1999Mete (Ankara,Turkey)Why does the zero longitude pass through Greenwich? whats the special about it?In 1884 delegates from 25 nations met in Washington D.C. to create a uniform starting line for time. Greenwich, England was chosen as zero degrees longitude because it was home to the world's primary telescope and observatory. Zero degrees longitude i s commonly known as the Prime Meridian.
May 6, 1999Aaron (Colorado, USA)What is the smallest political division in the world?(in area)The smallest country in the world is Tuvalu, a group of 9 islands in the Pacific Ocean. The total area of all the islands is 10 square miles. Not very big at all!
March 30, 1999Teresa (New York, USA)I want to know about Leonardo DaVinci. Period of time he lived,country and what made him famous in history.Here is our page about Leonardo da Vinci, Renaissance Man.
February 4, 1999Justin (Ohio, USA)When I was 9 I signed a thing that said my name would be sent to Mars. Is that true?Probably! 932,816 names were sent to Mars on January 3, 1999.
November 25, 1998Robert (Wisconsin, USA)How do I find my longitude and lattitude?The easiest way to find out is to look at an atlas. Find where you live, then look for the nearest north and south (up and down) line to get longitude and the nearest east to west (side-to-side) line to get latitude. Although you may not live right on a line, you can estimate your latitude and longitude. Latitude and longitude may also be included on a map of your city.
July 11, 1998Cheryl (California, USA)I was wondering when was the first year of B.C. Do you think the ending of time will happen the same year as the beginning year of B.C. only in A.D.?I see your reasoning. It certainly would be nice and neat, symmetrical even! Unfortunately, no one really knows the exact beginning year of B.C.! Some scientists would say it started over 2 million years ago when the first Homo habilis is dated to (the first undisputeable human). However, many religious groups would disagree. For instance, the Christian religion suggests that B.C. (when the first humans were created) started around the year 4000 B.C. This is a hot topic you're asking about Chery l!
June 26, 1998Laura (United Kingdom)In what order do the scientific headings go? e.g. Hypothesis, Method, etc. . .?It sounds like you're probably talking about the headings for a scientific paper. In that case, the order is: 1) Abstract - a brief summary of the entire paper, 2) Introduction - background information about the scientific problem and statement of the hypothesis, 3) Methods - a description of how the research and experiments were done, 4) Results - presentation of the actual data in the form of observations, measurments, numbers and statistics, 5) Discussion - in this section of the paper, the scientist must address the relationship between his or her hypothesis and the outcome of the experiment. Was the hypothesis supported? Why or why not? 6) Conclusion - a very brief summary of the conclusions that can be drawn as a result of the research and proposal of new hypotheses that can be tested in the future.
June 26, 1998Breona (California, USA)How do humans benefit from mosquitoes?I'll bet the last time you were bitten by one you probably thought they were good for nothing! Well, from a human's perspective, you're just about right. Mosquitoes can spoil an otherwise pleasant summer evening by their persistant harassment of us in an effort to suck our blood. And in some parts of the world, mosquitoes can be dangerous because they can carry deadly diseases, like malaria. But did you know that only female mosquitoes bite? Male mosquitoes sip nectar from flowers and are completely uninterested in your bare skin. In fact, some species of plants, including an endangered orchid, depend on mosquitoes for their pollination. Mosquitoes are also important to some animal species as food. Frogs, lizards, birds and bats are all quite fond of mosquitoes for snacking. In fact, a single little brown bat can catch and eat 600 mosquitoes an hour! My advice: if you don't like mosquitoes, make friends with a bat!!
June 22, 1998Laura (England)On a scientific report, what kind of graph(s) are mainly used?I've mainly used scatter-plots in the scientific reports that I've written. This is where you plot points of two sets of related data. Then you can fit a function to that plot (a line, a parabola, etc.) depending on what fits best. Scatter plots lea d to easy statistical and error analysis. I have seen other graphs used though - bar graphs, pie charts, etc.
April 7, 1998Maggie (Norway) Is it possible for us humans to make any use of black matter?So far, the existance of dark matter is only a theory. We would have to prove its existence before making any use of it!
March 3, 1998Jackson County Middle School Science Class(Kentucky, USA) Can two skydivers hear each other talking when they are traveling to earth?During the actual dive, when the divers are in freefall, (parachute unopened) the wind is much too loud for them to talk to each other. After the chutes have been opened, speed of falling is much less, so the wind is less, and the divers are able to converse.
February 19, 1998Timothy (England) Why are Maps of the earth drawn with north at the top?The people who first started drawing maps were the ancient Greeks. Because they lived in the northern hemisphere, the north star made a convenient reference point. Everyone could find it, and once it was found, you could figure out what direction yo u needed to go. So, since the north star was the first thing you needed to find, North was placed at the top of the map.
January 20, 1998Eric (Virginia, USA)What is the geometric relationship between the Great Pyramids in Egypt and our universe?Apparently, some of the pyramids were laid out in a manner corresponding to constellations such as Orion's belt. Others seem to match stars important to particular Pharaohs or dynasties.
November 26, 1997Sarah (Colorado, USA)Have you ever received any messages from or about other people or things on other planets? Do you think we are alone, is there any proof?None of us here at Windows to the Universe has ever received any messages from anyone from another planet! But to learn more about the possibility of life elsewhere in the universe, check out question #2 under "Alive and Living" and the corresponding link to our page about the Martian meteorite, ALH 84001.
November 26, 1997Christopher (New Jersey, USA)Are there any other body senses other than the senses of touch, smell, sight, taste and hearing?Nope--you've got them all. Some people may talk about a "sixth sense" which refers to a sense of "intuition" or "E.S.P." (extra-sensory perception), but this has not been scientifically proven. The known senses have specialized areas in the brain which receive information from nerves throughout the body which allow us to experience each type of "sensation". Until we find an area in the brain for psychic ability, we'll stick to having only 5 senses.

Windows to the Universe, a project of the National Earth Science Teachers Association, is sponsored in part is sponsored in part through grants from federal agencies (NASA and NOAA), and partnerships with affiliated organizations, including the American Geophysical Union, the Howard Hughes Medical Institute, the Earth System Information Partnership, the American Meteorological Society, the National Center for Science Education, and TERC. The American Geophysical Union and the American Geosciences Institute are Windows to the Universe Founding Partners. NESTA welcomes new Institutional Affiliates in support of our ongoing programs, as well as collaborations on new projects. Contact NESTA for more information. NASA ESIP NCSE HHMI AGU AGI AMS NOAA